Difference between revisions of "1967 AHSME Problems/Problem 26"

(Solution)
(Solution)
Line 13: Line 13:
 
<math>\log (1024) > 3</math>     
 
<math>\log (1024) > 3</math>     
  
<math>10 * log (2) > 3</math>
+
<math>10 * \log (2) > 3</math>
  
and <math>log (2) > 3/10</math>.
+
and <math>\log (2) > 3/10</math>.
  
  
Similarly, <math>8192 < 10000</math>, so <math>log (8192) < 4</math>
+
Similarly, <math>8192 < 10000</math>, so <math>\log (8192) < 4</math>
  
<math>13 * log (2) < 4</math>
+
<math>13 * \log (2) < 4</math>
  
and <math>log (2) < 4/13</math>
+
and <math>\log (2) < 4/13</math>
  
  
Therefore  <math>3/10 < log 2 < 4/13</math>  
+
Therefore  <math>3/10 < \log 2 < 4/13</math>  
 
so the answer is <math>\fbox{C}</math>
 
so the answer is <math>\fbox{C}</math>
  

Revision as of 19:23, 10 March 2017

Problem

If one uses only the tabular information $10^3=1000$, $10^4=10,000$, $2^{10}=1024$, $2^{11}=2048$, $2^{12}=4096$, $2^{13}=8192$, then the strongest statement one can make for $\log_{10}{2}$ is that it lies between:

$\textbf{(A)}\ \frac{3}{10} \; \text{and} \; \frac{4}{11}\qquad \textbf{(B)}\ \frac{3}{10} \; \text{and} \; \frac{4}{12}\qquad \textbf{(C)}\ \frac{3}{10} \; \text{and} \; \frac{4}{13}\qquad \textbf{(D)}\ \frac{3}{10} \; \text{and} \; \frac{40}{132}\qquad \textbf{(E)}\ \frac{3}{11} \; \text{and} \; \frac{40}{132}$

Solution

Since $1024$ is greater than $1000$.

$\log (1024) > 3$

$10 * \log (2) > 3$

and $\log (2) > 3/10$.


Similarly, $8192 < 10000$, so $\log (8192) < 4$

$13 * \log (2) < 4$

and $\log (2) < 4/13$


Therefore $3/10 < \log 2 < 4/13$ so the answer is $\fbox{C}$

See also

1967 AHSME (ProblemsAnswer KeyResources)
Preceded by
Problem 25
Followed by
Problem 27
1 2 3 4 5 6 7 8 9 10 11 12 13 14 15 16 17 18 19 20 21 22 23 24 25 26 27 28 29 30
All AHSME Problems and Solutions

The problems on this page are copyrighted by the Mathematical Association of America's American Mathematics Competitions. AMC logo.png